when providing flow required to reach the capacity of the pumping apparatus, floating strainers may be used for water as shallow as: select one: a. 3 inches (75 mm) b. 6 inches (150 mm) c. 8 inches (200 mm) d. 1 foot (300 mm)

Answers

Answer 1

Floating strainers can be used to provide the flow required to reach the capacity of a pumping apparatus in shallow water.

Among the given options, the minimum depth for using floating strainers is typically 6 inches (150 mm) (option b).

This depth allows for efficient operation without risking damage or drawing in debris that could hinder the pumping process. When it comes to providing the necessary flow required to reach the capacity of the pumping apparatus, floating strainers can be a useful tool.

These strainers are designed to filter out debris and prevent clogs, ensuring that the water being pumped is clean and free-flowing. However, the effectiveness of a floating strainer is limited by the depth of the water it is operating in.

According to the question, the options for the shallowest depth of water that a floating strainer can be used in range from 3 inches (75 mm) to 1 foot (300 mm).

It's important to note that the shallower the water, the more difficult it may be for a floating strainer to maintain a steady flow. This is because the intake of the strainer may be too close to the bottom of the water source, making it harder for the strainer to draw in water.



In general, it's recommended to use a floating strainer that is appropriate for the depth of water you will be pumping from. This means that if you're working with a water source that is only a few inches deep,

you should opt for a strainer that is designed to work in shallower depths. This will help ensure that the strainer is able to effectively filter out debris and maintain a steady flow of water, which is crucial when using a pumping apparatus.


In conclusion, the answer to the question is a. 3 inches (75 mm) - a floating strainer can be used for water sources as shallow as 3 inches (75 mm),

but it's important to choose a strainer that is appropriate for the To of water you will be working with to ensure that it can maintain a steady flow and effectively filter out debris.

To know more about depth click here

brainly.com/question/29198000

#SPJ11


Related Questions

A person walks 5. 0 kilometers north, then 5. 0 kilometers east. His displacement is closest to.

Answers

When a person walks 5.0 kilometers north, they are moving in a straight line towards the north direction.

If they then walk 5.0 kilometers east, they are moving in a straight line towards the east direction. To find the person's displacement, we need to calculate the straight-line distance between their starting point and their ending point.

We can use the Pythagorean theorem to do this. If we draw a right-angled triangle with the northward distance as the vertical leg and the eastward distance as the horizontal leg,

The hypotenuse of the triangle will be the straight-line distance between the starting and ending points.

Using the Pythagorean theorem, we can calculate the hypotenuse as follows: hypotenuse^2 = (northward distance)^2 + (eastward distance)^2, hypotenuse^2 = (5.0 km)^2 + (5.0 km)^2, hypotenuse^2 = 50 km^2, hypotenuse = sqrt(50) km.


hypotenuse = 7.07 km (rounded to two decimal places), Therefore, the person's displacement is closest to 7.07 kilometers. This means that the straight-line distance between their starting and ending points is 7.07 kilometers, even though they walked a total distance of 10 kilometers.



The displacement can be found by using the Pythagorean theorem, as it represents the shortest distance between the initial and final points of the person's journey. In this case, we have a right triangle with legs of 5.0 kilometers north and 5.0 kilometers east.



Applying the Pythagorean theorem, we get:

Displacement^2 = (5.0 km)^2 + (5.0 km)^2
Displacement^2 = 25 km^2 + 25 km^2
Displacement^2 = 50 km^2


To find the displacement, take the square root:
Displacement ≈ √50 km ≈ 7.07 km
So, the person's displacement is closest to 7.07 kilometers in the northeast direction.

To know more about theorem click here

brainly.com/question/30242664

#SPJ11

the radius of a right circular cone is increasing at a rate of 1.8 in/s while its height is decreasing at a rate of 2.6 in/s. at what rate is the volume of the cone changing when the radius is 150 in. and the height is 128 in.? incorrect: your answer is incorrect. in3/s

Answers

The volume of the cone is changing at a rate of approximately -3368.49 cubic inches per second. The negative sign indicates that the volume is decreasing.

To find the rate at which the volume of the cone is changing, we need to use related rates and the formula for the volume of a cone, which is V = (1/3)πr²h, where V is the volume, r is the radius, and h is the height.

Given that the radius is increasing at a rate of 1.8 in/s (dr/dt = 1.8) and the height is decreasing at a rate of 2.6 in/s (dh/dt = -2.6), we need to find dV/dt when r = 150 in and h = 128 in.

First, differentiate the volume formula with respect to time (t):
dV/dt = d(1/3πr²h)/dt

Apply the product rule and chain rule:
dV/dt = (1/3)π[2rh(dr/dt) + r²(dh/dt)]

Now, substitute the given values:
dV/dt = (1/3)π[2(150)(128)(1.8) + (150)²(-2.6)]

Perform the calculations:
dV/dt ≈ (1/3)π[55296 - 58500]

dV/dt ≈ (1/3)π[-3204]

dV/dt ≈ -3368.49 in³/s

Learn more about chain rule here:

brainly.com/question/28972262

#SPJ11

Please help me, the reward of 20 points.

Answers

Answer:

The coordinates of the vertices of the original triangle are (-4, 2), (2, 4), and

(-2, -2).

To obtain the new triangle, subtract 2 from each x-coordinate, and subtract 4 from each y-coordinate. So the coordinates of the vertices of the new triangle are (-6, -2), (0, 0) and (-4, -6). Draw the new triangle.

A man travels 108 km at a constant speed and finds that the journey would have taken 4 1/2 hours less if he had travelled at a speed 2 km/h faster. What was his speed? Provide a full explanation and work out. THANKSSSSS ;D

Answers

The speed of the man in the question is: 6 km/hr

How to find the speed from distance and time?

The formula to find the average speed when given distance and time is expressed as:

Average Speed = Distance/Time

Let the speed of the man be x.

At this speed(X), the total time he takes to travel 108km is 108/X.

Now, If he had travelled 2km/hour faster, his speed would have been X + 2 km/hour.

At this speed, he could have arrived at the destination 4.5 hours earlier.

This means that the time taken to travel would have been (108/x) - 4.5 hours if his speed had been (X + 2) km/hour.

So, according to the question, we have:

(108/X) - 4.5 = 108/ (X + 2) ----------- (1)

Solving this equation, we get X = 6 or -4. So, his speed is 6km/hour.

Read more about speed at: https://brainly.com/question/4931057

#SPJ1

Answer the following for the given figure. Please help answer A and B!

Answers

a. The congruent angles to angle 1 are given as follows: <7, <3 and <5.

b. The supplementary angles to angle 1 are given as follows: <2, <8, <4 and <6.

How to obtain the angles?

Two angles are classified as congruent when they have the same angle measure.

Hence the congruent angles to angle 1 are given as follows:

<7 -> opposite by the same vertex to 1.<3 -> corresponding to < 1.<5 -> corresponding to <7.

Two angles are called supplementary when the sum of their measures is of 180º, hence the supplementary angles to angle 1 are given as follows:

<2 -> linear pair with <1.<8 -> linear pair with <1.<4 -> corresponding with < 2.<6 -> corresponding with <8.

More can be learned about angle measures at https://brainly.com/question/25716982

#SPJ1

PART B If the survey has a margin of
error of 2.5%, what is the difference
between the minimum and maximum
estimates of the total number of
students at the university that use
online tutoring services?
A.60
B.121
C.504
D.1007

Answers

The range of possible values for the difference between the minimum and maximum estimates is: 500.

Therefore, the answer is C. 504.

To find the difference between the minimum and maximum estimates, we need to calculate the range of the possible values.

The margin of error is 2.5%, which means that the actual value could be 2.5% higher or lower than the estimated value.
Let's call the estimated value of the total number of students who use online tutoring services "x."

Then the minimum estimate would be 0.975x (x minus 2.5%) and the maximum estimate would be 1.025x (x plus 2.5%).
So the difference between the minimum and maximum estimates is:
1.025x - 0.975x = 0.05x
We don't know the value of x, but we do know that it's between 2,000 and 12,000.

Therefore, the range of possible values for the difference between the minimum and maximum estimates is:
0.05(12,000 - 2,000) = 500.

The answer is C. 504.

For similar question on possible values.

https://brainly.com/question/26261370

#SPJ11

Rewrite as an exponential equation.
In 2=y

Answers

Answer:

[tex]y = ln(2) [/tex]

[tex] {e}^{y} = 2[/tex]

Let X have possible values {1, 2, 3, 4, 5} and probability mass function
x 1 2 3 4 5
px(x) 1/7 1/14 3/14 2/7 2/7
(a) Calculate P(X ≤ 3). (b) Calculate P(X < 3). (c) Calculate P(X < 4. 12 | X > 1. 638)

Answers

The probability of P(X ≤ 3) is 2/7, the probability of P(X < 3) is 3/14, the probability of P(X < 4. 12 | X > 1. 638) is 7/22.

To calculate P(X ≤ 3), we need to add the probabilities of all outcomes less than or equal to 3

P(X ≤ 3) = P(X = 1) + P(X = 2) + P(X = 3)

= 1/7 + 1/14 + 3/14

= 4/14

= 2/7

To calculate P(X < 3), we need to add the probabilities of all outcomes strictly less than 3

P(X < 3) = P(X = 1) + P(X = 2)

= 1/7 + 1/14

= 3/14

To calculate P(X < 4 | X > 1.638), we first need to find the probability of X > 1.638

P(X > 1.638) = P(X = 2) + P(X = 3) + P(X = 4) + P(X = 5)

= 1/14 + 3/14 + 2/7 + 2/7

= 11/14

Then, we can use the formula for conditional probability

P(X < 4 | X > 1.638) = P(X < 4 and X > 1.638) / P(X > 1.638)

We need to calculate the probability of X < 4 and X > 1.638, which is the probability of X = 2 or X = 3

P(X < 4 and X > 1.638) = P(X = 2 or X = 3)

= P(X = 2) + P(X = 3)

= 1/14 + 3/14

= 1/4

Therefore,

P(X < 4 | X > 1.638) = (1/4) / (11/14)

= 7/22

To know more about probability here

https://brainly.com/question/30861635

#SPJ4

the following code segment is intended to set max equal to the maximum value among the integer variables x, y, and z. the code segment does not work as intended in all cases.

Answers

we do not have the code segment, we cannot identify which of the cases above will cause the code segment to not work as intended.

What is a conditional statement?

A conditional statement is a type of structure to express the relationship between two dependent variables. Its structure is as follows:

The code segment in question uses variables and conditional statements to determine the maximum value among three integer variables x, y, and z. However, there are cases where this code segment may not work as intended.

To determine which initial values for x, y, and z will cause this issue, we need to look at the code segment itself.

The code segment likely involves setting a variable called "max" to the value of one of the three variables (x, y, or z), and then using conditional statements to check if the other variables are larger than the current value of "max".

If they are, the value of "max" is updated to that variable.

To determine which initial values will cause issues, we need to consider cases where the conditional statements will not work as intended.

For example, if all three variables have the same value, the code segment may not correctly identify the maximum value.

Similarly, if two variables have the same value, but that value is smaller than the third variable, the code segment may not correctly identify the maximum value.

Based on the answer choices provided, it appears that the code segment may not work as intended for the initial values x = 1, y = 3, z = 2.

In this case, the initial value of "max" would be set to 1, and the conditional statements would not update the value of "max" to the correct maximum value of 3.

In conclusion, the code segment may not work as intended in cases where there are ties or when the initial values are not in order. It is important to test the code segment with different initial values to ensure that it works correctly in all cases.

The code segment in question is intended to set the variable "max" equal to the maximum value among the integer variables x, y, and z.

To demonstrate that the code segment does not work as intended in all cases, we need to evaluate each of the given initial values for x, y, and z.

1. x = 1, y = 2, z = 3: In this case, the maximum value is 3, which corresponds to the variable z.

2. x = 1, y = 3, z = 2: In this case, the maximum value is 3, which corresponds to the variable y.

3. x = 2, y = 3, z = 1: In this case, the maximum value is 3, which corresponds to the variable y.

4. x = 3, y = 2, z = 1: In this case, the maximum value is 3, which corresponds to the variable x.

Since we do not have the code segment, we cannot identify which of the cases above will cause the code segment to not work as intended.

However, you can test each case by implementing the code segment, assigning the initial values to the variables x, y, and z, and then checking if the variable "max" is set to the correct maximum value.

If any of the cases result in an incorrect "max" value, it means that the code segment does not work as intended for that particular set of initial values.

Hence, we do not have the code segment, we cannot identify which of the cases above will cause the code segment to not work as intended.

To learn more about the conditional statement visit: brainly.com/question/18152035

#SPJ4

the probability that a given eighty year-old person will die in the next year is 0.27. what is the probability that exacly 10 out or as eighty-year-olds will die in the next year (a) 0.8615 (b) 0.4685 (c) 0.1385 (d) 0.1208 (e)0.0000000031795

Answers

the probability that exactly 10 out of 10 eighty-year-olds will die in the next year is approximately 0.0000000031795, which is closest to option (e).

The number of deaths in a group of 80-year-olds follows a binomial distribution with parameters n = 10 and p = 0.27. The probability mass function of this distribution is given by:

P(X = k) = (n choose k) * p^k * (1-p)^(n-k)

where X is the random variable denoting the number of deaths, k is the number of deaths, (n choose k) is the binomial coefficient "n choose k" which represents the number of ways to choose k items out of n without order and is given by:

(n choose k) = n! / (k! * (n-k)!)

where ! denotes the factorial operation.

Substituting n = 10 and p = 0.27, we get:

P(X = 10) = (10 choose 10) * 0.27^10 * (1-0.27)^(10-10) = 0.0000000031795

To learn more about binomial visit:

brainly.com/question/13870395

#SPJ11

Magan invested $790 in an account paying an interest rate of 83% compounded
quarterly. Angel invested $790 in an account paying an interest rate of 8%
compounded continuously. After 7 years, how much more money would Magan have
in his account than Angel, to the nearest dollar?

Answers

Answer:3

Step-by-step explanation:

1437.9874-1444.882=3.1054

Help pls and thank you

Answers

The closest value of t for the right triangle is 16.5 ft

The correct answer is an option (a)

Let us assume that in the attached diagram of right triangle the angle A measures 60  degrees.

Here, the hypotenuse of right triangle measures 19 ft.

We know that in right triangle, the sine of angle θ is nothing but the ratio of opposite side of angle θ to the hypotenuse.

Consider the sine of angle A

sin(A) = opposite side of angle A / hypotenuse

sin(60°) = t / 19

We know that from the standard trigonometric table the value of sin(60°) is [tex]\frac{\sqrt{3} }{2 }[/tex]

Substitute this value in above equation we get,

[tex]\frac{\sqrt{3} }{2 }[/tex]=  t/19

We solve this equation to find the value of t.

t = 19 ×  [tex]\frac{\sqrt{3} }{2 }[/tex]

t = 16.45 ft.

t ≈ 16.5 ft.

Therefore, the correct answer is an option (a)

Learn more about the sine of angle here:

https://brainly.com/question/3827723

#SPJ1

Use the integral test to determine whether the series is convergent or divergent.

Answers

We need to find the function f(n) whose terms are the same as the series in question. We can then integrate this function from n=1 to infinity and determine if the integral is convergent or divergent. If it is convergent, then the series is convergent. If it is divergent, then the series is also divergent.

To determine whether a series is convergent or divergent using the integral test, we need to first check if the series satisfies three conditions:

1) The terms of the series are positive.

2) The terms of the series are decreasing.

3) The series has an infinite number of terms.

Assuming these conditions are satisfied, we can use the integral test which states that if the integral of the function f(x) from n=1 to infinity is convergent, then the series with terms a_n = f(n) is also convergent. Conversely, if the integral is divergent, then the series is also divergent.

Learn more about integral here:

brainly.com/question/18125359

#SPJ11

a model used for the yield Y of an agricultural crop as a function of the nitrogen level n in the soil (measured in appropriate units) isY = kN / 36+N^2where k is a positive constrant. What nitrogen level gives the best yield?

Answers

The nitrogen level that gives the best yield function is where the optimal nitrogen level is 6 units.

To find the nitrogen level that gives the best yield, we need to find the maximum value of the yield function Y. To do this, we can take the derivative of Y with respect to N, set it equal to zero, and solve for N.

dy/dN = k(36 - N²)/(36 + N²)²

Setting dy/dN equal to zero, we get:

0 = k(36 - N²)/(36 + N²)²

Solving for N, we get:

N = ±6

Since N has to be a positive value, the optimal nitrogen level is N = 6 units.

Learn more about functions:

https://brainly.com/question/28628175

#SPJ4

On Thursday night Antonio watched a movie that was 1 hour and 43 minutes long. If the movie ended at the time shown on the clock below, what time did Antonio start watching the movie? Be sure to include a.m. or p.m. in your answer.

Answers

Antonio started watching the movie at 5:17pm

At what time did Antonio start the movie?

If the movie ended at 7:00pm, and we know that the movie was 1 hour and 43 minutes long, we can then subtract 1 hour and 43 minutes from 7:00pm to find out what time Antonio started the movie.

To subtract 1 hour and 43 minutes from 7:00pm:

We will first convert 7:00pm to 24-hour format which is 19:00.We can subtract 1 hour and 43 minutes which gives us:

= 19:00 - 1:43

= 17:17

As a 24-hour mode, when we convert 17:17 to 12 hours, this gives us 5:17pm.

Note: The movie ended at 7:00pm

Read more about time

brainly.com/question/26862717

#SPJ1

What is the probability of selecting a heart replacing then selecting a star?

Answers

The probability of selecting a heart and then a star with replacement is approximately 0.1875 or 18.75%.

Assuming that a standard deck of 52 playing cards is used, with 13 cards of each suit (including hearts) and 4 suits in total, the probability of selecting a heart on the first draw and then selecting a star (presumably meaning a card from a different suit) on the second draw with replacement is

P (heart than star)

= P (heart) × P (star)

= 13/52 × 39/52

= 507/2704

= 0.1875

where P (heart) is 13/52 is the probability of selecting a heart on the first draw (since there are 13 hearts in the deck), and P (star) is the probability of selecting a card that is not a heart on the second draw (since there are 39 non-heart cards left in the deck after the heart is replaced).

Therefore, the probability of selecting a heart and then a star with replacement is approximately 0.1875 or 18.75%.

To know more about probability here

https://brainly.com/question/30034780

#SPJ4

-- The given question is incomplete, the complete question

"What is the probability of selecting a heart and then a star with replacement?" --

Kiran is thinking about building a structure like this for his younger cousins to play on.
The entire structure is made out of soft foam so the children don't hurt themselves.
How much foam-would-Kiran need to build this play structure?
Kiran would need?
cubic inches of foam to build this play structure.
The entire structure is covered with vinyl so it is easy to wipe clean.
How much vinyl would Kiran need to cover this play structure?
Kiran would need?
square inches of vinyl to cover this play structure.
The foam costs 0.008¢ per in³.
The vinyl costs 0.006€ per in².
What is the total cost for all the foam and vinyl needed to build this play structure?
The cost for the foam needed is $
The cost for the vinyl needed is $
The total cost for all the foam and vinyl needed to build this play structure is $

Answers

The total cost for all the foam and vinyl needed to build the play structure would be $419.91

How to calculate the cost

Let's say the play structure is a cube with each side measuring 36 inches. The total volume of the foam needed to build this cube would be:

V = (36 in)³ = 46,656 in³

Since the structure is a cube, the surface area would be:

A = 6 × (36 in)² = 7,776 in²

The cost for the foam would be:

46,656 in³ × $0.008/in³ = $373.25

The cost for the vinyl would be:

7,776 in² × $0.006/in² = $46.66

Total cost will be:

$373.25 + $46.66 = $419.91

Learn more about cost on

https://brainly.com/question/25109150

#SPJ1

heights for a certain age are normally distributed, with a mean of 40 inches and a standard deviation of 5 inches. use the empirical rule to find the lower and upper boundaries for the middle 68% of heights.

Answers

The lower and upper boundaries for the middle 68% of heights falls between 35 inches and 45 inches.

The empirical rule, too known as the 68-95-99.7 rule, could be a statistical rule that depicts the approximate rate of perceptions that fall within a certain number of standard deviations from the mean of a normal distribution. Particularly, the empirical rule states that for a normal distribution, around:

68% of the observations fall under one standard deviation of the mean.95% of the observations fall under two standard deviations of the mean.99.7% of the observations fall under three standard deviations of the mean.

According to the Empirical rule,

Mean = 40 inches(given)

Standard deviation = 5 inches(given)

To find out the lower and upper boundaries for the middle of 68% heights-

lower boundary = mean - standard deviation

upper boundary = mean + standard deviation

So,

lower boundary = 40 - 5 = 35 inches.

upper boundary = 40 + 5 = 45 inches.

Therefore, 68% of heights fall between 35 inches and 45 inches.

To learn more about empirical rule,

https://brainly.com/question/30573266

A news organization interested in chronicling winter holiday travel trends conducted a survey. Of the 96 people surveyed in the eastern half of a country, 42 said they fly to visit family members for the winter holidays. Of the 108 people surveyed in the western half of the country, 81 said they fly to visit family members for the winter holidays.
Use a calculator to construct a 99% confidence interval for the difference in population proportions of people in the eastern half of a country who fly to visit family members for the winter holidays and people in the western half of a country who fly to visit family members for the winter holidays. Assume that random samples are obtained and the samples are independent.
Round your answers to three decimal places.

Answers

We are 99% confident that the true difference in population proportions of people in the eastern half of the country who fly to visit family members for the winter holidays and people in the western half of the country who fly to visit family members for the winter holidays is somewhere between -0.422 and -0.114.

Next, we need to calculate the standard error of the difference in sample proportions. This gives us an idea of how much the sample difference in proportions can be expected to vary from the true population difference in proportions. We use the following formula to calculate the standard error:

√((p₁(1-p₁)/n₁)+(p₂(1-p₂)/n₂))

where p₁ and p₂ are the sample proportions, and n₁ and n₂ are the sample sizes. Plugging in the values we have, we get a standard error of 0.094.

Now that we have the sample proportions and the standard error, we can use a confidence interval formula to calculate the range of values that we can be confident contains the true population difference in proportions. For a 99% confidence interval, the formula is:

(sample proportion 1 - sample proportion 2) +/- (critical value x standard error)

The critical value is obtained from a t-distribution table, with degrees of freedom equal to the smaller of (n1-1) and (n2-1). For a 99% confidence level and 44 degrees of freedom, the critical value is 2.689.

Plugging in the values we have, we get a confidence interval of:

0.438 - 0.75 +/- 2.689 x 0.094

= -0.422 to -0.114

To know more about confidence interval here

https://brainly.com/question/24131141

#SPJ4

Use the information given in the table on the right to complete each of the following statements. Brenda is 50 inches tall. Her z-score is .

Answers

The z-score of Brenda for the mean of 49 inches and standard deviation 2 inches is equal to 0.5.

Mean is equal to 49 inches

Standard deviation is equal to 2 inches

Brenda is 50 inches tall.

To find Brenda's z-score,

Calculate the number of standard deviations that her height is away from the mean height for 7-year-olds.

z-score = (Brenda's height - Mean height) / Standard deviation

Substituting the given values, we get,

⇒ z-score = (50 - 49) / 2

⇒ z-score = 0.5

Statement 1,

Brenda's height is 0.5 standard deviations above the mean height for 7-year-olds.

Statement 2,

Approximately 68.27% of 7-year-olds are shorter than Brenda.

Using a standard normal distribution table to find the percentage of the area under the curve to the left of z = 0.5.

Therefore, Brenda's z-score for the given mean and standard deviation  is equal to 0.5.

Learn more about z-score here

brainly.com/question/12615610

#SPJ4

The above question is incomplete, the complete question is:

Use the information given in the table on the right to complete each of the following statements. Brenda is 50 inches tall. Her z-score is

Find individual value in normal distribution

Age           Mean                 Standard deviation

7years       49 inches             2 inches

approximately what percentage of u.s. adults is overweight? select one: a. 24 percent b. 44 percent c. 64 percent d. 84 percent

Answers

According to the Centers for Disease Control and Prevention (CDC), about 73.6 percent of adults in the United States are overweight or obese. So, the correct answer is D).

This means that a majority of adults in the U.S. have a body mass index (BMI) above the healthy range, which is defined as a BMI of 18.5-24.9, which puts them at increased risk for a range of health issues, including heart disease, diabetes, and some types of cancer.

The prevalence of overweight and obesity has been increasing in the United States in recent decades, and it is a significant public health concern. Obesity is a complex issue with many causes, including genetics, environment, and lifestyle factors such as diet and physical activity levels. So, the correct answer is D).

To know more about overweight:

https://brainly.com/question/29847112

#SPJ4

--The given question is incomplete, the complete question is given

"  According to the Centers for Disease Control and Prevention (CDC), approximately what percentage of u.s. adults is overweight? select one: a. 24 percent b. 44 percent c. 64 percent d. 73.6 percent "--

The three characteristics required to properly describe a sampling distribution are _____
✓ 1. mean 2. variance 3. shape

Answers

The three characteristics required to properly describe a sampling distribution are mean, variance, and shape.

1. Mean: The mean, also known as the average, represents the central tendency of the sampling distribution. It is calculated by adding up all the sample means and dividing the sum by the total number of samples.

2. Variance: The variance is a measure of how much the sample means vary from the overall mean of the sampling distribution. It helps determine the spread or dispersion of the data.

3. Shape: The shape of a sampling distribution describes its general appearance. Common shapes include symmetrical (e.g., normal distribution) and skewed (e.g., positively or negatively skewed) distributions. The shape can give insights into the underlying population and the behavior of the data.

By considering the mean, variance, and shape, you can effectively describe the characteristics of a sampling distribution.

Learn more about  quantity variance here brainly.com/question/14116780

#SPJ11

Find 95% confidence interval for the average number of sick days an employee will take per year, given the employee is 47 .

Answers

For sample of employee’s age and the number of sick days the employee takes per year, the 95% confidence interval for the average number of sick days an employee will take per year, the 47 employee is equals to the (0.81, 6.81).

The estimated regression line for model of number of sick days the employee takes per year days is Sick Days = 14.310162 − 0.2369(Age)

Prediction for avg no. of sick days for employee aged 47, [tex]\bar X = 14.310162 - 0.2369 × Age[/tex]

= 14.310162 - 0.2369 × 47

= 3.175862 = 3

Sample size, n = 10

Sample error, SE = 1.682207

So, standard deviations, s =

[tex]SE× \sqrt{n} = 1.682207 × \sqrt{10}[/tex] = 5.31960

Number of degree of freedom, df = 10 - 1 = 9

Level of significance, α = 0.05 and α/2 = 0.025

Based on the provided information, the critical value for α = 0.05 and df = 9 ( degree of freedom) is equals to the 2.262. Now, the 95% confidence interval is written as, [tex]CI = \bar X ± \frac{ t_c × s}{\sqrt{n}}[/tex].

Substitutes all known values in above formula, [tex]CI = 3 ± \frac{ 2.262 × 5.31960}{\sqrt{10}}[/tex]

= 3 ± 3.805152234

=> CI = (0.81, 6.81)

Hence, required confidence interval is (0.81, 6.81).

For more information about confidence interval, visit:

https://brainly.com/question/17097944

#SPJ4

Complete question:

The above figure complete the question.

The personnel director of a large hospital is interested in determining the relationship (if any) between an employee’s age and the number of sick days the employee takes per year. The director randomly selects ten employees and records their age and the number of sick days which they took in the previous year. The estimated regression line and the standard error are given.

Sick Days=14.310162−0.2369(Age)

se = 1.682207

Find the 95% confidence interval for the average number of sick days an employee will take per year, given the employee is 47. Round your answer to two decimal places

Help pls and thank you

Answers

If right triangle BAC, m∠A = 90, m∠B = 45, and AC = 8, the length of BC is 4√2 units. So, correct option is A.

In a right triangle, the side opposite to the 90-degree angle is called the hypotenuse, and the other two sides are called the legs. In triangle BAC, AC is the hypotenuse and AB and BC are the legs. We are given that AC = 8 and ∠B = 45 degrees. We can use trigonometric ratios to find the length of BC.

The trigonometric ratio for the sine of an angle is the ratio of the length of the opposite side to the length of the hypotenuse. The trigonometric ratio for the cosine of an angle is the ratio of the length of the adjacent side to the length of the hypotenuse. Since we know that ∠B = 45 degrees, we can use the fact that sin(45) = cos(45) = 1/√2.

Let x be the length of BC. Then, we have:

sin(45) = x/8

x/8 = 1/√2

x = 8/√2

We can simplify this expression by rationalizing the denominator:

x = 8/√2 * √2/√2

x = 8√2/2

x = 4√2

So, correct option is A.

To learn more about triangle click on,

https://brainly.com/question/31437939

#SPJ1

help meeee thank you dont guess pleaseeee

Answers

The right unit multipliers is A, 24 ft²/1 min . 12 in/1 ft . 12 in/1 ft . 1 min/60 sec.

How to solve unit multipliers?

The correct choice is:

24 ft²/1 min . 12 in/1 ft . 12 in/1 ft . 1 min/60 sec.

This correctly uses unit multipliers to convert 24 square feet per minute to square inches per second.

To convert from square feet to square inches, multiply by the conversion factor (12 inches per 1 foot) twice because of dealing with area.

To convert from minutes to seconds, multiply by the conversion factor (1 minute per 60 seconds).

So, when multiplied these conversion factors together with the given value of 24 ft²/1 min:

24 ft²/1 min . 12 in/1 ft . 12 in/1 ft . 1 min/60 sec

= (24 x 12 x 12) in² / (1 x 1 x 1) min x (1 x 1 x 60) sec

= 4,608 in²/sec

Therefore, the answer is 24 ft²/1 min . 12 in/1 ft . 12 in/1 ft . 1 min/60 sec.

Find out more on unit multipliers here: https://brainly.com/question/13736702

#SPJ1

(2) five cards are dealt from a standard 52-card deck. (a) how many such hands have only black cards? (b) how many such hands have a full house of aces and fives (3 aces and 2 fives)?g

Answers

The total number of hands with a full house of aces and fives is 36 * 10 = 360.



(a) To find the number of hands with only black cards:
Step 1: There are 26 black cards in a standard 52-card deck (13 spades and 13 clubs).
Step 2: We need to choose 5 black cards from the 26 available.
Step 3: Use the combination formula: C(n, k) = n! / (k!(n-k)!) where n is the total number of items and k is the number of items we want to choose.
Step 4: Calculate C(26, 5) = 26! / (5!(26-5)!) = 26! / (5!21!) = 65,780.

Answer (a): There are 65,780 hands with only black cards.

(b) To find the number of hands with a full house of aces and fives:
Step 1: There are 4 aces and 4 fives in a standard 52-card deck.
Step 2: Choose 3 aces from the 4 available (C(4, 3)).
Step 3: Choose 2 fives from the 4 available (C(4, 2)).
Step 4: Multiply the combinations from steps 2 and 3: C(4, 3) * C(4, 2).
Step 5: Calculate C(4, 3) = 4! / (3!(4-3)!) = 4.
Step 6: Calculate C(4, 2) = 4! / (2!(4-2)!) = 6.
Step 7: Multiply the results from steps 5 and 6: 4 * 6 = 24.

Answer (b): There are 24 hands with a full house of aces and fives.

to learn more about number click here:

brainly.com/question/30752681

#SPJ11

In a research study on trends in marriage and​ family, 2​% of randomly selected parents said that they never spank their children. The​ 95% confidence interval is from 1.2​% to 2.8​% ​(n=1241​).
a) Interpret the interval in this context
A. One is 95% confident that if one were to ask every parent, between 1.2% to 2.8% of them would say they never spank their children
B. There is a 95% chance that, if one were to ask every parent, 2% of them would say they never spank their children
C. There is a 95% chance that, if one were to ask overy parent, between 1.2% to 2.8% of them would say they never spank their children
D. One is 95% confident that of the 1236 parents polled between 12% 10 2.8% of them would say they never spank their children

Answers

The given information presents the results of a research study on trends in marriage and family, which indicates that 2% of randomly selected parents said they never spank their children.

The 95% confidence interval for this proportion is from 1.2% to 2.8%, based on a sample size of 1241.

A. One is 95% confident that if one were to ask every parent, between 1.2% to 2.8% of them would say they never spank their children.

The 95% confidence interval means that if we were to take many random samples of parents and construct a 95% confidence interval for each sample, then about 95% of those intervals would contain the true proportion of parents who never spank their children.

In other words, we can be 95% confident that the true proportion of parents who never spank their children is somewhere between 1.2% and 2.8%.

Option (C) is the correct interpretation of the interval in this context, which states that there is a 95% chance that, if one were to ask every parent, between 1.2% to 2.8% of them would say they never spank their children.

This implies that the true proportion of parents who never spank their children is likely to fall between 1.2% and 2.8%.

Option (A) is also a correct interpretation of the interval, as it implies that we are 95% confident that the proportion of parents who never spank their children falls between 1.2% and 2.8%.

Option (B) is incorrect because the 95% confidence interval refers to the range of possible values for the true proportion of parents who never spank their children, not the probability that a randomly selected parent would say they never spank their children.

Option (D) is incorrect because the sample size is 1241, not 1236.

In summary, the 95% confidence interval of 1.2% to 2.8% for the proportion of parents who never spank their children indicates that we are 95% confident that the true proportion of parents who never spank their children falls between these values, if we were to ask every parent.

For similar question on probability.

https://brainly.com/question/30535529

#SPJ11

(L7) a=3 cm, b=√12.96 cm, c=4 cmThe triangle is a(n) _____ triangle.

Answers

The triangle with side lengths a=3 cm, b=√12.96 cm, and c=4 cm can be classified as a scalene triangle, as all three sides have different lengths.

To classify the triangle based on its side lengths, we need to compare the lengths of the three sides. In this case, side a has a length of 3 cm, side b has a length of √12.96 cm, and side c has a length of 4 cm.

A scalene triangle is a triangle in which all three sides have different lengths. In this scenario, since the lengths of sides a, b, and c are different, the triangle can be classified as a scalene triangle.

It is important to note that the triangle's angles can also be used to classify triangles. However, since only the side lengths are provided in this question, we can determine the triangle's classification based solely on the side lengths, which leads us to conclude that it is a scalene triangle.

to learn more about scalene triangle click here:

brainly.com/question/30765574

#SPJ11

find the average rate of change of over the interval . for how many values of in the interval does the instantaneous rate of change of equal the average rate of change of over that interval?

Answers

there are two values of x in the interval where the instantaneous rate of change of is equal to the average rate of change of over the interval.

To find the average rate of change of over the interval , we need to calculate the slope of the line passing through the two endpoints of the interval.

The slope of the line passing through the points and is given by:

( - )/( - ) = ( -3 - 3)/(1 - (-1)) = -6/2 = -3

Therefore, the average rate of change of over the interval is -3.

To find how many values of in the interval have instantaneous rate of change equal to the average rate of change, we need to find the derivative of :

f'(x) = 3x^2 - 3x - 3

Setting f'(x) equal to the average rate of change, we get:

3x^2 - 3x - 3 = -3

Simplifying the equation, we get:

3x^2 - 3x = 0

Factoring out 3x, we get:

3x(x - 1) = 0

Therefore, the solutions are x = 0 and x = 1.

To learn more about solutions visit:

brainly.com/question/30665317

#SPJ11

A steamer goes downstream and covers the distance between two ports in 4 hours while it covers the same distance upstream in 5 hours. If the speed of the stream is 2 km per hour, find the speed of the steamer in still water

Answers

The speed of the streamer in still water to cover the same distance in 4 hours for downstream and 5 hours in upstream in equal to 18km/hour.

Time taken by streamer in downstream to cover some distance = 4hours

Time taken by streamer in upstream to cover same distance = 5 hours

Let the speed of the streamer in still water be x km/hour.

Speed of the stream is 2 km per hour

Then ,

Speed of the streamer in downstream = ( x+ 2) km/hour

Speed of the streamer in upstream = ( x - 2) km/hour

Distance covered by streamer in down stream in 4 hours

= Distance covered by streamer in up stream in 5 hours

⇒ 4 ( x + 2) = 5( x -2)

⇒ 4x + 8 = 5x -10

⇒ 5x - 4x = 10 + 8

⇒ x = 18 km/hour.

Therefore, the speed of the streamer in still water in equal to 18km/hour.

Learn more about speed here

brainly.com/question/24147771

#SPJ4

Other Questions
What is the enthalpy change for the following reaction when 3.30 moles of oxygen react with excess methane?CH4(g) + 2O2(g) CO2(g) + 2H2O(g) H = -802 kJa. -486.1 kJ b. -1203.0 kJ c. -1323.3 kJ d. -1604.0 kJ e. -2646.6 kJ light of wavelength 409 nm in air undergoes constructive interference when it is perpendicularly reflected from an oil spill on a plastic sheet. the refractive index of the oil is 1.2 and that of the plastic is 1.48. what is the main argument of the passage? what claims does the author make to support the argument? how valid, relevant, and sufficient is the reasoning and evidence used to support the argument and claims? does the author use false statements or fallacious reasoning to support the argument and claims? use evidence from the text to support your response. your evaluation of the argument should be two or three complete paragraphs. what are examples of epigenetic chemical markers associated with dna that determines when, where and by how much genes are expressed in each cell? g draw two normal curves that have the same mean but different standard deviations. describe the similarities and differences. tolerance has the following effect on intoxication: a. high tolerance causes intoxication faster b. high tolerance causes intoxication slower c. tolerance has no effect on intoxication d. low tolerance causes intoxication faster e. low tolerance causes intoxication slower the tax was implemented to make sure that taxpayers who were generating income pay some income tax, rather than disproportionately benefiting from tax advantaged items.T/F The Doctor, Amy, & Rory are trapped in a hotel of horrors & they can't find the Tardis. The Doctor has to save all that he can while dealing with his own worst fear.This the summary for _________________. What is recounted by the various readings at the easter vigil?. Your non dominant hemisphere houses what abilities? The writer wants to replace the underlined text in sentence 12 (reproduced below) with a more formal and precise phrase.Moreover, once Shakespeare moved to London as a young man, he would have been exposed to a wealth of ideas and knowledge, which, good for us, he used to create some of the greatest works of literature the world has ever known.Which of the following phrases would best accomplish this goal? The likelihood individuals have of sharing in the opportunities and benefits of society. For the reaction aA products, select the reaction order(s) that best fit(s) the observations. The half-life is constant. A manager decides not to do a background or criminal record check on Jim before hiring him. Jim is hired and a few weeks later he assaults a customer. It turns out that Jim had a criminal record for assault. In light of this information ____ has probably occurred Which of the following is the correct first four terms of the geometric progression with initial term 3 and common ratio 1/2?A. 3, 6, 12, 24B. 3, 3/2, 3/4, 3/8C. 2/3, 4/3, 8/3, 16/3D. 3/2, 314, 3/8, 3/16E. 1/2, 3/2, 9/2, 2712 According to the interactionist perspective,Gabby would most likely face prejudice atLansing and Brett when white lawyers ________with her.A) have more contactB) competeC) make friendsD) cooperate galileo's discovery of four moons orbiting jupiter showed that planetary bodies could move and carry moons. this supports the model of the universe presented by which of the following? What is the best next step in managament of a 70year old man with prostate Ca with a gleason score of 6, PSA Self-fulfilling prophecies in the realm of psychology are referred to as:. a significant impediment to e-commerce has been the questionable enforceability of contracts entered into through electronic means, such as the internet or e-mail, because of the writing requirements under contract and sales law. T/F ?